2
$\begingroup$

Is the following true?

Let $X$ and $Y$ be separable Banach spaces and consider their dual spaces $X^*$ and $Y^*$ equipped with weak* topology. Suppose that a linear map $T:X^*\to Y^*$ is sequentially continuous. Then, is it true that $T$ is continuous?

I suspect this may be true using the following two facts:

(i) Any closed ball in $X^*$ (and in $Y^*$) is metrizable as $X$ and $Y$ are separable.

(ii) A convex set in $X^*$ (and in $Y^*$) is weak* closed iff it is sequentially closed (by Krein-Smulian theorem).

From (i), we have that $T$, when restricted to a closed ball, is continuous. But I am not able to go further from this.

Any help or hint would be appreciated. Thank you.

$\endgroup$
3
  • $\begingroup$ You should specify what you mean by continuity of $T$. Should $T$ be continuous with respect to the weak topologies? $\endgroup$ Oct 24, 2017 at 10:39
  • $\begingroup$ I have already said that $X^*$ and $Y^*$ are equipped with weak* topology. So everything above is with respect to these topology, be it the continuity of $T$ or the metrizability of closed balls of $X^*$. $\endgroup$ Oct 24, 2017 at 11:29
  • $\begingroup$ This is Corollary 4.14 in my book. $\endgroup$
    – Nik Weaver
    Oct 24, 2017 at 18:04

1 Answer 1

4
$\begingroup$

This is true. To show it, in the following I will use $\langle \mbox{-}, \mbox{-} \rangle$ for the pairing between a space and its dual (with the vectors from the space on the left, the dual on the right).

For each $y \in Y$, if we define $f(y) = \langle y, \mbox{-}\rangle \circ T$, then $f(y)$ is a sequentially continuous linear functional on $X^*$. Its kernel is therefore a sequentially closed convex set, so is weak-* closed in $X^*$ (by, according to taste, Krein-Šmulian or Banach-Dieudonne or Grothendieck's completeness theorem), and therefore $f(y)$ is actually weak-* continuous. Therefore there exists a $g(y) \in X$ such that $\langle g(y), \mbox{-}\rangle = f(y)$. It is then not difficult to show that $g(y)$ is a linear map $Y \rightarrow X$ such that $\langle g(y), \phi \rangle = \langle y, T(\phi) \rangle$ for all $y \in Y$ and $\phi \in X^*$. This implies that $T$ is continuous from the weak-* of $X^*$ to the weak-* of $Y^*$ (not hard to prove directly, but a reference is Schaefer's Topological Vector Spaces IV.2.1).

Note, however, that the analogous statement for incomplete normed spaces is false -- any element $x$ of the completion of a normed space $X$ that is not part of $X$ defines a linear functional that is weak-* continuous on the ball of $X^*$ but not on $X^*$.

$\endgroup$
5
  • $\begingroup$ did you use the separability anywhere? it seems this holds without separability? $\endgroup$
    – DJA
    Nov 12, 2019 at 22:45
  • $\begingroup$ @DJA I use the separability to go from sequentially closed to weak-* closed. What I'm using is the fact that the weak-* topology on bounded subsets of the dual of a separable Banach space is metrizable, so the intersection of the kernel of $f(y)$ with each closed bounded set is closed. Krein-Šmulian/Banach-Dieudonné can then be applied to deduce that the kernel of $f(y)$ is weak-* closed. $\endgroup$ Nov 13, 2019 at 17:46
  • $\begingroup$ @DJA It is not true in general that sequential continuity implies continuity. There is a nice counterexample in Solovay's model where all sets are Lebesgue measurable: the linear map $\int : \ell^\infty([0,1]) \rightarrow \mathbb{R}$ defined by Lebesgue integration is sequentially continuous with $\ell^\infty([0,1])$ given the weak-* topology as the dual of $\ell^1([0,1])$, essentially by the dominated convergence theorem, but not continuous. $\endgroup$ Nov 13, 2019 at 17:56
  • $\begingroup$ @DJA An example that works in ordinary set theory is to take $X = \aleph_1$ with the order topology. Then by the Riesz representation theorem and the fact that $X$ is a scattered space, $C_0(X)^* \cong \ell^1(X)$, which defines a weak-* topology on $\ell^1(X)$. Then the mapping defined by summation $\ell^1(X) \rightarrow \mathbb{R}$ is sequentially continuous but not continuous. $\endgroup$ Nov 13, 2019 at 18:01
  • $\begingroup$ That is extremely helpful, thank you very much! $\endgroup$
    – DJA
    Nov 13, 2019 at 18:21

Your Answer

By clicking “Post Your Answer”, you agree to our terms of service and acknowledge you have read our privacy policy.

Not the answer you're looking for? Browse other questions tagged or ask your own question.